You are on page 1of 123

QUANT FOUNDATION BATCH

For IBPS CLERK PRE - 2020

Starting From
5 October 2020
[12:15 PM]
on
Use Code ‘’AVP10’’ for 10% Discount

Click Here to Join


IBPS CLERK PRE - 2020
QUANT FOUNDATION BATCH
Starting From 5 October 2020
Timing 12 : 15 PM

FOLLOW ME ON UNACADEMY “ARUN SINGH RAWAT” FOR FREE CLASSES

Use Code ‘’AVP10’’ for 10% Discount


Click Here to Join
ARUN SINGH RAWAT
Practice Course on RRB PO/Clerk
Mains Batch - 2020
Starting From 28 Sept. 2020
Timing 02 : 00 PM

FOLLOW ME ON UNACADEMY “ARUN SINGH RAWAT” FOR FREE CLASSES

Use Code ‘’AVP10’’ for 10% Discount


Click Here to Join
ARUN SINGH RAWAT
ARUN SINGH RAWAT
More than 7 Year Experience in offline and online coaching
Highest viewership on every banking Platform

Arun Singh Rawat ( WiFiStudy & Unacademy Bank Educator )

ARUN SINGH RAWAT


Highest viewership on every banking Platform More than 100 videos on YouTube Trending list
STEP - 02
यहा Search करे STEP - 03
STEP - 01 unacademy Click on this
Play Store
Open करे

ARUN SINGH RAWAT


STEP - 05
Login with Your your
Mobile Number
STEP - 04
Install

ARUN SINGH RAWAT


ARUN SINGH RAWAT
AVP10 AVP10 AVP10

ARUN SINGH RAWAT


Upcoming batch – IBPS CLERK & RRB MAINS 2020

IBPS CLERK PRE IBPS RRB PO/CLERK


FOUNDATION MAINS
BATCH BATCH –
12 : 15 PM 2 : 00 PM

Starting from 5 October 2020 Starting from 28 sep 2020

USE CODE AVP10 for 10% discount


ARUN SINGH RAWAT
Follow

ARUN SINGH RAWAT


Running Paid Course for IBPS CLERK PRE & RRB MAINS BATCH

Foundations course for IBPS


CLERK PRE 2020 IBPS RRB PO & CLERK MAINS

12 : 15 pm 2 : 00 PM

ARUN SINGH RAWAT


After Getting Subscriptions – How to Join

How to download Pdf of the session


ARUN SINGH RAWAT
Watch Daily Free Class on Unacademy at 5 : 00 pm

ARUN SINGH RAWAT


Give me your valuable Feedback on Unacademy

ARUN SINGH RAWAT


Maths by Arun Singh Rawat sir
ARUN SINGH RAWAT
ARUN SINGH RAWAT
After end of class give your feedback

ARUN SINGH RAWAT


Click Here toRAWAT
ARUN SINGH Join
Click Here
ARUN SINGH RAWAT to Join
Click Here to Join
ARUN SINGH RAWAT
Click Here to
ARUN SINGH Join
RAWAT
ARUN SINGH RAWAT
FOLLOW ON UNACADEMY “ARUN SINGH RAWAT” FOR FREE CLASSES
Top
बिलकुल ऐसा ही आएगा | 5050 Arithmetic
Number SeriesQuestions
| 5 Marks पक्के करो
384 ml of mixture contains milk and water in the ratio of 7:5
respectively. 25% of mixture is taken out and ‘x’ ml of milk and 80 ml of
water is added into the remaining mixture such that quantity of milk in
final mixture becomes 50%. Find the value of ‘x’.
384 ml मिश्रण िें दूध और पानी का अनुपात क्रिशः 7:5 है ।25% मिश्रण का मनकाला
जाता है और शेष मिश्रण िें ’x 'ml दूध और 80 ml पानी मिलाया जाता है , मजससे
अंमति मिश्रण िें दूध की िात्रा 50% हो जाती है । ‘x’ का िान ज्ञात करें ?
54 ml
32 ml
48 ml
36 ml
None of these
Quantity of milk in 384 ml of mixture = 7/12 × 384 = 224 ml
Quantity of water in 384 ml of mixture = 384 – 224 = 160 ml
According to question;
0.75 × 224 + x = 0.75 × 160 + 80
Or, 168 + x = 120 + 80
Or, x = 200 – 168
Or, x = 32 ml
Hence, option b.
FOLLOW ON UNACADEMY “ARUN SINGH RAWAT” FOR FREE CLASSES
Top
बिलकुल ऐसा ही आएगा | 5050 Arithmetic
Number SeriesQuestions
| 5 Marks पक्के करो
Sam invested Rs.X at 15% p.a. at SI for 5 years. If total amount received
by Sam at the end of 5 years is Rs.84000, then find value of (X + 2000).
सैि ने 5 वषष के मलए साधारण ब्याज पर 15% की वामषषक दर पर X रुपये का मनवेश
मकया। यमद सैि को 5 वषों के अंत िें प्राप्त कुल रामश 84000 रुपये है , तो (X +
2000) का िान ज्ञात कीमजए।
40000
60000
50000
80000
70000
Solution:
ATQ
𝑋×15×5
= 84000 − 𝑋
100
𝑋 = 48000
Required value = 48000 + 2000
= 50000 Rs.
FOLLOW ON UNACADEMY “ARUN SINGH RAWAT” FOR FREE CLASSES
Top
बिलकुल ऐसा ही आएगा | 5050 Arithmetic
Number SeriesQuestions
| 5 Marks पक्के करो
The ratio of speed of boat in still water and speed of stream is 13:5
respectively. If boat covers a certain distance in 6 hrs 15 mins in
downstream, then find the time (approx.) taken by the boat to cover
the same distance in upstream?
शांत जल िें नाव की गमत का प्रवाह की गमत से अनुपात क्रिश: 13:5 है । यमद नाव को
प्रवाह के अनुकूल एक मनमित दूरी तय करने के मलए 6 घंटे 15 मिनट का सिय लगता
है , तो नाव को प्रवाह के प्रमतकूल सिान दूरी को तय करने िें मकतना सिय (लगभग)
लगेगा?
10 hr
14 hr
16 hr
8 hr
12 hr
Let speed of boat in still water and speed of stream be 13x km/hr. and 5x km/hr. respectively
So, Downstream speed of boat: upstream speed of boat = (13x+5x) :(13x-5x)
= 9 : 4 or (9a:4a (let))
ATQ, let time taken by boat to cover the same distance in upstream be t hour.
9a×6.25=4a×t
t = 14.0625 hr. = 14 hr (approx.)
FOLLOW ON UNACADEMY “ARUN SINGH RAWAT” FOR FREE CLASSES
Top
बिलकुल ऐसा ही आएगा | 5050 Arithmetic
Number SeriesQuestions
| 5 Marks पक्के करो
P and Q together can complete the whole work in 27(3/11) days while R
alone can complete the same work in 36 days. If ratio of efficiency of R
and P is 5: 3, then Q alone can complete the whole work in how many
days?
एक साथ P और Q 27(3/11) मदनों िें पूरे कायष को पूणष कर सकते हैं , जबमक R
अकेले सिान कायष को 36 मदनों िें पूणष कर सकती है । यमद R और P की कायष-क्षिता
का अनुपात 5 : 3 है , तो Q अकेले पूरे कायष को मकतने मदनों िें पूणष कर सकती है ?
(a) 30 days
(b) 60 days
(c) 45 days
(d) 40 days
(e) 50 days
P alone can complete the whole work in = 36 * 5/3 = 60 days
So, Q alone can complete the whole work in = 1/(11/300 - 1/60) = 50 days
FOLLOW ON UNACADEMY “ARUN SINGH RAWAT” FOR FREE CLASSES
Top
बिलकुल ऐसा ही आएगा | 5050 Arithmetic
Number SeriesQuestions
| 5 Marks पक्के करो
P and Q started a business by investing Rs. 4500 and Rs. 5000
respectively. Q left the business at the end of 5 months but rejoined 4
months later. At the end of the year, the difference between the shares
received by Q and P out of the total profit was Rs. 910. What was the
total profit received at the end of the year?
P और Q ने क्रिशः 4500 और 5000 रुपये का मनवेश करके एक व्यवसाय शुरू
मकया। Q ने 5 िहीने के अंत िें व्यवसाय छोड़ मदया लेमकन 4 िहीने बाद मिर से
शामिल हुआ। वषष के अंत िें , कुल लाभ िें से Q और P द्वारा प्राप्त महस्ों के बीच का
अंतर 910 रुपये है । वषष के अंत िें प्राप्त कुल लाभ मकतना है ?
(a) Rs. 6120

(b) Rs. 6210

(c) Rs. 6240

(d) Rs. 6110

(e) Rs. 6410


Q was out of business for 4 months, i.e., he was in the business for 8 months.
Ratio of investments of P and Q = (4500 * 12):(5000 * 8) = 27:20
So, difference in their shares = (27 - 20)/47 * total profit = 910
So, total profit = Rs. 6110
FOLLOW ON UNACADEMY “ARUN SINGH RAWAT” FOR FREE CLASSES
Top
बिलकुल ऐसा ही आएगा | 5050 Arithmetic
Number SeriesQuestions
| 5 Marks पक्के करो
Population of a town is 1500 out of which 8% are unemployed adults.
Adult to non-adult population are in the ratio 11: 4 respectively. If 65%
adults are employed males, then how many adult females are
employed?
एक शहर की जनसंख्या 1500 है , मजसिें से 8% वयस्क बेरोजगार हैं । वयस्क और
गैर-वयस्क जनसंख्या का अनुपात क्रिशः 11 : 4 है । यमद 65% वयस्क पु रुष हैं , तो
मकतनी वयस्क िमहलाओं को रोजगार मिला है ?
(a)265
(b)250
(c)235
(d)240
(e)225
Total population = 1500
Unemployed adult population = 8% of 1500 = 120
Adult population = 1500 x 11/15 = 1100
Employed adult male population = 65% of 1100 = 715
Employed adult population = 1100 - 120 = 980
Then, employed adult female population = 980 - 715 = 265
FOLLOW ON UNACADEMY “ARUN SINGH RAWAT” FOR FREE CLASSES
Top
बिलकुल ऐसा ही आएगा | 5050 Arithmetic
Number SeriesQuestions
| 5 Marks पक्के करो
Average of present ages of A and B is 38 years. Age of A after 33 years
will be 30% more than age of B after 6 years. Find present age of A.
A और B की वतषिान आयु का औसत 38 वषष है । 33 वषष के बाद A की आयु 6 वषष के
बाद B की आयु से 30% अमधक होगी।A की वतषिान आयु ज्ञात करें |
44 years
32 years
36 years
34 years
None of these
Let present age of B is ‘x’ years
Present age of A = 38 × 2 – x = ’76 –x’ years
According to question;
(76 – x + 33) = 1.30 × (x + 6)
Or, 109 – x = 1.3x + 7.8
Or, 2.3x = 101.2
Or, x = 44
Present age of A = 76 – 44 = 32 years
FOLLOW ON UNACADEMY “ARUN SINGH RAWAT” FOR FREE CLASSES
Top
बिलकुल ऐसा ही आएगा | 5050 Arithmetic
Number SeriesQuestions
| 5 Marks पक्के करो
A shopkeeper sold a table at Rs. 10350 and made a profit of 15% and
sold a chair at Rs. 8100 and made a loss of 10%. If he earned overall 20%
of profit while selling both table and chair, then what will be the total
selling price of table and chair?
एक दुकानदार ने एक िेज़ 10,350 रु. की बेची और 15% का लाभ किाया और एक
कुसी 8100 रु. की बेची और 10% की हामन उठाई। यमद उसने िेज़ और कुसी दोनों
को बेचकर कुल 20% का लाभ अमजषत मकया, तो िेज़ और कुसी का कुल मवक्रय िूल्य
मकतना होगा?
(a) Rs. 21540
(b) Rs. 21450
(c) Rs. 21160
(d) Rs. 21600
(e) Rs. 26100
Sol.
CP of table = 10350 * 100/115 = Rs. 9000
CP of chair = 8100 * 100/90 = Rs. 9000
So, total selling price = (9000 + 9000) * 120/100 = 18000 *
1.2 = Rs. 21600
FOLLOW ON UNACADEMY “ARUN SINGH RAWAT” FOR FREE CLASSES
Top
बिलकुल ऐसा ही आएगा | 5050 Arithmetic
Number SeriesQuestions
| 5 Marks पक्के करो
Speed of a boat in downstream is 20% more than speed of boat in still
water. If the boat can travel a distance of 144 km upstream and 144 km
downstream in 20 hours. Find the distance travelled by boat in still
water in 8 hours.
धारा अनुप्रवाह िें नाव की गमत शांत जल िें नाव की गमत से 20% अमधक है । यमद
नाव 20 hours िें 144 km की धारा मवरुद्ध और 144 km धारा अनुप्रवाह की दूरी को
तय कर सकती है ।8 hours िें शां त जल िें नाव द्वारा तय की गयी दूरी मकतनी है ?
120 km
128 km
112 km
160 km
None of these
Let speed of boat in still water is ‘x’ km/h
Speed of boat in downstream = 1.20 × x = 1.2x km/h
Speed of boat in upstream = x – 0.2x = 0.8 km/h
According to question;
144/0.8x + 144/1.2x = 20
Or, 180/x + 120/x = 20
Or, x = 300/20 = 15
Desired distance = 15 × 8 = 120 km
FOLLOW ON UNACADEMY “ARUN SINGH RAWAT” FOR FREE CLASSES
Top
बिलकुल ऐसा ही आएगा | 5050 Arithmetic
Number SeriesQuestions
| 5 Marks पक्के करो
Sam took a loan of Rs. 3600 from a bank and time for which he took the
loan is one-eighth of the rate of simple interest on which he took the
loan. If interest paid by him is Rs. 4608, then find the rate of simple
interest.
सैि ने एक बैंक से Rs. 3600 का ऋण मलया और मजस सिय के मलए उसने ऋण मलया
था वह उस साधारण ब्याज की दर का एक-आठवां था मजस पर उसने ऋण मलया था।
यमद उसके द्वारा भुगतान मकया गया ब्याज Rs. 4608 है , मिर साधारण ब्याज की दर
मकतनी है ?
40%
32%
16%
24%
None of these
Solution
Let the rate is 8x%
So, time = 1/8 × 8x = x years
According to question;
(3600 × x × 8x)/100 = 4608
Or, x2 = 16
So, x = 4
Desired rate = 4 × 8 = 32%
Hence, option b.
FOLLOW ON UNACADEMY “ARUN SINGH RAWAT” FOR FREE CLASSES
Top
बिलकुल ऐसा ही आएगा | 5050 Arithmetic
Number SeriesQuestions
| 5 Marks पक्के करो
A and B entered into a business with initial investment of Rs. 19800 and
Rs. 15300, respectively and the ratio of time for which they made their
investment is 9:11 respectively. If profit share of A is Rs. 14400, then
find the total profit earned by A and B together.
A और B ने क्रिशः Rs. 19800 और Rs. 15300 के शुरुआती मनवेश के साथ एक
व्यवसाय िें प्रवेश मकया और उनके मनवेश की अवमध का अनुपात क्रिशः 9:11 था।
यमद A के लाभ का महस्ा Rs. 14400 है , मिर A और B द्वारा अमजषत कुल लाभ
मकतना है ?
Rs. 28000
Rs. 25600
Rs. 30400
Rs. 24000
None of these
Ratio of profit share of A to B = (19800 × 9):(15300 × 11) = 18:17
Desired profit = 35/18 × 14400 = Rs. 28000
Hence, option a.
FOLLOW ON UNACADEMY “ARUN SINGH RAWAT” FOR FREE CLASSES
Top
बिलकुल ऐसा ही आएगा | 5050 Arithmetic
Number SeriesQuestions
| 5 Marks पक्के करो
Sam has some amount and he invested the money in two schemes P and Q in the ratio
of 3 : 5 respectively. Scheme P offers compound interest at the rate of 12.5% p.a. for
two years and scheme Q offers simple interest at the rate of 15% p.a. for two years. If
difference between the interest earned from both the schemes is Rs. 1406.25, then find
how much money was invested in scheme Q by Sam?
सैि के पास कुछ रामश है और वह दो योजनाओं P और Q िें क्रिशः 3: 5 के अनुपात िें रामश का मनवेश
करता है । योजना P पर 2 वषष के मलए 12.5% की वामषषक दर से चक्रवृद्धद्ध ब्याज प्रस्तामवत है तथा योजन
Q पर 15% की वामषषक दर से साधारण ब्याज प्रस्तामवत है । यमद दोनों योजनाओं से अमजषत ब्याज के
बीच का अंतर 1406.25 रुपये है , तो सैि द्वारा योजना Q िें मनवेश की गई रामश ज्ञात कीमजये।
Rs. 10000
Rs. 12000
Rs. 8000
Rs. 16000
Rs. 20000
Solution:
Let the total investment of Sam = 800X
3
So, investment of Sam in scheme P = 800X × 8 = 300𝑋
5
And, investment of Sam in scheme Q = 800𝑋 × 8 = 500𝑋
ATQ
2 9 9
500𝑋 × 15 × − 300𝑋 × × − 300𝑋 = 1406.25
100 8 8
150𝑋 − 79.6875𝑋 = 1406.25
𝑋 = 20 Rs,
Amount investment in scheme Q by Sam = 500 × 20 = 1000 Rs.
FOLLOW ON UNACADEMY “ARUN SINGH RAWAT” FOR FREE CLASSES
Top
बिलकुल ऐसा ही आएगा | 5050 Arithmetic
Number SeriesQuestions
| 5 Marks पक्के करो
If ratio of circumference of bigger circle to that of smaller circle is 8:3
and sum of radius of both circles is 77cm, then find area of bigger circle.
यमद छोटे वृत्त की तुलना िें बड़े वृत्त की पररमध का अनुपात 8: 3 है और दोनों वृत्तों की
मत्रज्या का योग 77 सेिी है , तो बड़े वृत्त का क्षेत्रिल ज्ञात कीमजए।
6342 𝒄𝒎𝟐
7682 𝒄𝒎𝟐
8468 𝒄𝒎𝟐
5462 𝒄𝒎𝟐
9856 𝒄𝒎𝟐
Solution:
Let radius of bigger circle and smaller circle be R cm and r
cm respectively.
ATQ
22
2× ×𝑅 8
7
22 =
2× ×𝑟 3
7
𝑅 8
=3
𝑟
Let R & r be 8a & 3a respectively.
8𝑎 + 3𝑎 = 77
𝑎 = 7 𝑐𝑚
22
Required area = × 7×8 2
7
= 9856 𝑐𝑚2
FOLLOW ON UNACADEMY “ARUN SINGH RAWAT” FOR FREE CLASSES
Top
बिलकुल ऐसा ही आएगा | 5050 Arithmetic
Number SeriesQuestions
| 5 Marks पक्के करो
A started a business by investing a certain amount. At the end of 7 months, A
left the business while B joined the business by investing Rs. 250 more than
that invested by A. At the end of the year, the shares of profit received by A
and B were Rs. 1600 and Rs. 1200 respectively. How much did B invest?
A ने एक मनमित धनरामश का मनवेश करके एक व्यवसाय शुरू मकया। 7 िहीने के अंत िें , A ने
व्यवसाय छोड़ मदया जबमक B, A द्वारा मनवेश मकए गए धनरामश से 250 रुपये अमधक का
मनवेश करके व्यवसाय िें शामिल हुआ। वषष के अंत िें , A और B को प्राप्त लाभ के शेयर
क्रिशः 1600 रुपये और 1200 रुपये हैं । B ने मकतना मनवेश मकया?
(a) Rs. 5250

(b) Rs. 5750

(c) Rs. 4250

(d) Rs. 6250

(e) Rs. 4750


Ratio of their shares of profit = 1600:1200 = 4:3
Hence, they must have invested in the ratio 4:3 respectively.
Let the amount invested by A be Rs. a and that by B be Rs. (a + 250).
So, 7a:5 * (a + 250) = 4:3
21a = 20a + 5000
a = 5000
Hence, B invested Rs. 5250.
FOLLOW ON UNACADEMY “ARUN SINGH RAWAT” FOR FREE CLASSES
Top
बिलकुल ऐसा ही आएगा | 5050 Arithmetic
Number SeriesQuestions
| 5 Marks पक्के करो
P alone can complete the whole work in 60 days while Q and R together
can complete the 16(2/3)% of the work in 4 days. If Ratio of efficiency
of Q and P is 5: 3, then in how many days P and R together can
complete the whole work?
अकेले P पूरे कायष को 60 मदनों िें पूणष कर सकती है जबमक एक साथ Q और R 4
मदनों िें कायष का 16(2/3)% पूणष कर सकती हैं । यमद Q और P की कायष-दक्षता का
अनुपात 5 : 3 है , तब मकतने मदनों िें एक साथ P और R कायष पूणष कर सकती हैं ?
(a) 31(7/11) days

(b) 34(6/11) days

(c) 32(8/11) days

(d) 33(7/11) days

(e) None of these


Q alone can complete the whole work in = 60 * 3/5 = 36 days
R alone can complete the whole work in = 1/(1/24 - 1/36) = 72 days
Hence, P and R together can complete the whole work in = 1/(1/72 + 1/60) = 32(8/11) days
FOLLOW ON UNACADEMY “ARUN SINGH RAWAT” FOR FREE CLASSES
Top
बिलकुल ऐसा ही आएगा | 5050 Arithmetic
Number SeriesQuestions
| 5 Marks पक्के करो
Sum of present ages of A and B is 38 years and sum of ages of B and C
after 4 years is 49 years. If the respective ratio of ages of A and C
before 3 years is 5: 6 respectively, then find the present age of A.
A और B की वतषिान आयु का योग 38 वषष है तथा 4 वषष के बाद B और C की आयु
का योग 49 वषष है । यमद 3 वषष से पहले A और C की आयु का संबंमधत अनुपात
क्रिशः 5: 6 है , तो A की वतषिान आयु ज्ञात ज्ञात करें ।
(a) 18 years
(b) 12 years
(c) 15 years
(d) 21 years
(e) 20 years
A + B = 38
A = 38 - B
(B + 4) + (C + 4) = 49
C = 41 - B
And, (A - 3): (C - 3) = 5: 6
(38 - B - 3): (41 - B - 3) = 5: 6
B = 20 years
Therefore, A = 38 - 20 = 18 years
FOLLOW ON UNACADEMY “ARUN SINGH RAWAT” FOR FREE CLASSES
Top
बिलकुल ऐसा ही आएगा | 5050 Arithmetic
Number SeriesQuestions
| 5 Marks पक्के करो
1408 balls are distributed among 80 children such that each boy
received 25% more than each girl. If number of girls are 50% more than
the number of boys, then how balls received by each girl?
1408 गेंदों को 80 बच्ों के िध्य िें इस प्रकार बााँटा जाता है मक प्रत्येक लड़के को
प्रत्येक लड़की से 25% अमधक गेंदें प्राप्त होती हैं । यमद लड़मकयों की संख्या लड़कों की
संख्या से 50% अमधक है , तब प्रत्येक लड़की को मकतनी गेंदें प्राप्त होती हैं ?
(a) 12
(b) 8
(c) 16
(d) 4
(e) 20
Total number of children = 80
Number of girls/number of boys = 150/100 = 3/2
Then, number of girls = 80 * 3/5 = 48
Number of boys = 80 - 48 = 32
Let number of balls received by each girl = n
Then, number of balls received by each boy = 125% of n =
5n/4
Now, 1408 = 48 * n + 32 * 5n/4
n = 16
FOLLOW ON UNACADEMY “ARUN SINGH RAWAT” FOR FREE CLASSES
Top
बिलकुल ऐसा ही आएगा | 5050 Arithmetic
Number SeriesQuestions
| 5 Marks पक्के करो
Sam spent 40% of monthly income on petrol, 20% of the remaining on
study, 60% of the remaining after spending on petrol and study, on
gaming and saved Rs. 1200. Find the monthly income of Sam.
सैि ने अपनी िामसक आय का 40% पेटरोल पर खचष मकया, शेष िें से 20% अध्ययन
पर, पेटरोल और अध्ययन पर खचष करने के बाद शेष िें से 60% गेमिंग पर खचष मकए
और Rs. 1200 की बचत की।सैि की िामसक आय मकतनी है ?
Rs. 6250
Rs. 7250
Rs. 6650
Rs. 6450
None of these
Let monthly income of Sam is Rs. 100x
According to question;
0.60 × 0.80 × 0.40 × 100x = 1200
Or, 19.2x = 1200
Or, x = 62.5
Monthly income of Sam = 62.5 × 100 = Rs. 6250
FOLLOW ON UNACADEMY “ARUN SINGH RAWAT” FOR FREE CLASSES
Top
बिलकुल ऐसा ही आएगा | 5050 Arithmetic
Number SeriesQuestions
| 5 Marks पक्के करो
Vessel P contains X liters of a mixture of oil and water in the ratio 7:3
respectively and vessel Q contains 200 liters of another mixture of oil
and water in the ratio 1:9 respectively. If the mixture of both vessels is
mixed and the quantity of oil and water becomes equal to each other in
the resulting mixture, then find X.
बतषन P िें क्रिशः 7: 3 के अनुपात िें तेल और पानी के मिश्रण का X लीटर है और
बतषन Q िें क्रिशः 1: 9 के अनुपात िें तेल और पानी का अन्य मिश्रण का 200 लीटर
है । यमद दोनों बतषन का मिश्रण मिलाया जाता है और पररणािी मिश्रण िें तेल और पानी
की िात्रा एक-दूसरे के बराबर हो जाती है , तो X ज्ञात कीमजए।
360
300
400
350
320
Solution:
ATQ
7𝑥 1 3𝑋 9
+ 200 × 10 = + 200 × 10
10 10
4𝑥
= 160
10
𝑋 = 400
FOLLOW ON UNACADEMY “ARUN SINGH RAWAT” FOR FREE CLASSES
Top
बिलकुल ऐसा ही आएगा | 5050 Arithmetic
Number SeriesQuestions
| 5 Marks पक्के करो
Average marks obtained by 55 students of a class are 72. But marks of
some of the students were misread as 84, 74, 70 and 88 in place of 56,
32, 48 and 70 respectively. Find the original average marks obtained by
the students.
एक कक्षा के 55 छात्रों द्वारा प्राप्त मकए गए औसत अंक 72 हैं । लेमकन कुछ छात्रों के
अंकों को 56, 32, 48 और 70 के स्थान पर क्रिशः 84, 74, 70 और 88 गलत पढ़
मलया गया था। छात्रों द्वारा प्राप्त वास्तमवक औसत अंक मकतने है ?
70
68
65
62
None of these
Total marks obtained by students = 72 × 55 = 3960
Total misread marks = (84 + 74 + 70 + 88) – (56 + 32 + 48 + 70) = 110
Desired Average = (3960 – 110)/55 = 70
FOLLOW ON UNACADEMY “ARUN SINGH RAWAT” FOR FREE CLASSES
Top
बिलकुल ऐसा ही आएगा | 5050 Arithmetic
Number SeriesQuestions
| 5 Marks पक्के करो
In a basket, there are 5 green ,4 blue and 2 red balls in a basket. What is
the probability of both balls being blue or green ball, if 2 balls are
picked from the basket?
एक टोकरी िें, 5 हरी, 4 नीली और 2 लाल गेंदें हैं । यमद टोकरी से 2 गेंद मनकाली
जाती है , तो दोनों गेंदों की नीली या हरी गेंद होने की प्रामयकता क्या है ?
16/55
21/55
39/55
34/55
None of these
Solution:
Here, there is 5 green, 4 blue and 2 red balls
5𝐶 + 4𝐶 10+6
Probability of both ball being blue of green = 2
11𝐶
2
=
2 55
16
=
55
FOLLOW ON UNACADEMY “ARUN SINGH RAWAT” FOR FREE CLASSES
Top
बिलकुल ऐसा ही आएगा | 5050 Arithmetic
Number SeriesQuestions
| 5 Marks पक्के करो
Speed of boat A and B in still water are 23 km/hr and 29 km/hr. If
stream is flowing with 5 km/hr, then find the difference between
upstream distance covered by boat A in 7 hours and boat B in 13 hours.
शांत जल िें नाव A और B की गमत 23 मकिी/घं और 29 मकिी/घं है । यमद नदी 5
मकिी/घं की गमत से प्रवामहत है , तब 7 घंटे िें नाव A और 13 घंटे िें नाव B द्वारा धारा
-प्रमतकूल मदशा िें तय की गई दूरी के िध्य का अंतर ज्ञात कीमजए।
(a) 172 km
(b) 180 km
(c) 176 km
(d) 188 km
(e) 186 km
Here, speed of boat A in still water = 23 km/hr
Speed of boat B in still water = 29 km/hr
Stream speed = 5 km/hr
Then, upstream speed of boat A = 23 - 5 = 18 km/hr
And, upstream speed of boat B = 29 - 5 = 24 km/hr
Therefore, difference between upstream distance covered by boat A in 7 hours and boat B in 13 hours
= 13 * 24 - 7 * 18
= 186 km
FOLLOW ON UNACADEMY “ARUN SINGH RAWAT” FOR FREE CLASSES
Top
बिलकुल ऐसा ही आएगा | 5050 Arithmetic
Number SeriesQuestions
| 5 Marks पक्के करो
Three cards are drawn randomly from a pack of 52 well-shuffled cards,
then find the probability of getting one ace card & two face cards?
52 सही से िेटें गए कार्ष के एक पैकेट से तीन कार्ष यादृद्धिक रुप से मनकाले जाते हैं ,
तो एक ऐस कार्ष और दो िेस कार्ष प्राप्त करने की प्रामयकता ज्ञात कीमजए।
7/2210
11/1105
91/5247
66/5525
7/340
Solution:
12𝐶 × 4𝐶
Required probability = 2
52𝐶
1
3
66
= 5525
FOLLOW ON UNACADEMY “ARUN SINGH RAWAT” FOR FREE CLASSES
Top
बिलकुल ऐसा ही आएगा | 5050 Arithmetic
Number SeriesQuestions
| 5 Marks पक्के करो
Ratio of volume of cylinder to that of cone is 10:3 and ratio of radius of
cylinder to that of cone is 2 : 3 and ratio of radius to height of cylinder
is 2 : 5. If height of cone is 14 cm, then find curved surface area of
cylinder.
बेलन के आयतन का शंकु के आयतन से अनुपात 10:3 है और बेलन की मत्रज्या का
शंकु की मत्रज्या से अनुपात 2 : 3 है तथा बेलन की मत्रज्या का ऊाँचाई से अनुपात 2 : 5
है । यमद शंकु की ऊाँचाई 14 सेिी है , तो बेलन का वक्र पृष्ठीय क्षेत्रिल ज्ञात कीमजए।
2720 cm²
3080 cm²
3140 cm²
3580 cm²
2920 cm²
Solution:
Let radius of cylinder & Cone be 2x cm & 3x cm respectively.
5
So height of cylinder = 2𝑥 × 2
= 5𝑥 cm
ATQ
22
× 2𝑥 2 ×5𝑥 10
7
22 1 =
× × 3𝑥 2 ×14 3
7 5

𝑥=7
22
Required area = 2 × × 2×7×5×7
7
= 3080 𝑐𝑚2
FOLLOW ON UNACADEMY “ARUN SINGH RAWAT” FOR FREE CLASSES
Top
बिलकुल ऐसा ही आएगा | 5050 Arithmetic
Number SeriesQuestions
| 5 Marks पक्के करो
Sam deposited Rs 25000 in SBI bank for 2 years which offers compound
interest of 10%. Due to some emergency, she withdrew Rs 20000 at the
end of 1st year. How much amount she will get at the end of 2nd year?
सैि ने 2 वषष के मलए एसबीआई बैंक िें 25,000 रु. जिा मकए थे, मजस पर 10% का
चक्रवृद्धद्ध ब्याज प्रदान मकया जाता दे ता है । कुछ आपात द्धस्थमत के कारण, उसने 1 वषष
के अंत िें 20,000 रु. मनकाल मलए। उसे 2 वषष के अंत िें मकतनी धनरामश मिलेगी?
(a) Rs. 7500
(b) Rs. 8500
(c) Rs. 8250
(d) Rs. 8350
(e) None of these
Total amount that Sam would have got at the end of 1st year = Rs (25000 X 1.1) = Rs 27500
Amount withdrawn at the end of 1st year = Rs 20000
Principal amount for the second year on which Sam will get interest at the end of 2nd year = (27500 - 20000) = Rs 7500
Hence total amount that Sam will get at the end of 2nd year = Rs (7500 X 1.1) = Rs 8250
FOLLOW ON UNACADEMY “ARUN SINGH RAWAT” FOR FREE CLASSES
Top
बिलकुल ऐसा ही आएगा | 5050 Arithmetic
Number SeriesQuestions
| 5 Marks पक्के करो
A shopkeeper bought rice A at a rate of Rs. 48 per kg and mixed it with
rice B which costs Rs. 64 per kg and sold the mixture for Rs. 52 per kg. If
total amount of mixture is 128 kg, then quantity of rice A is:
एक दुकानदार ने Rs. 48 प्रमत kg की दर से चावल A खरीदा और इसे चावल B के
साथ मिला मदया मजसका िूल्य Rs. 64 प्रमत kg है और इस मिश्रण को Rs. 52 प्रमत kg
िें बेचा। यमद मिश्रण की कुल िात्रा 128 kg है , तो चावल A की िात्रा है :
96 kg
32 kg
64 kg
16 kg
None of these
Desired quantity of rice A = 3/4 × 128 = 96 kg
FOLLOW ON UNACADEMY “ARUN SINGH RAWAT” FOR FREE CLASSES
Top
बिलकुल ऐसा ही आएगा | 5050 Arithmetic
Number SeriesQuestions
| 5 Marks पक्के करो
42 liters mixture of milk and water in the ratio of 4: 3 mixed with 18
liters mixture of milk and water in the ratio of 1: 2 and final mixture is
again mixed with 12 liters mixture of milk and water in the ratio of 1: 1.
What is the ratio of milk and water in the final mixture?
4: 3 के अनुपात वाले 42 लीटर दूध और पानी के मिश्रण को 1: 2 के अनुपात वाले 18
लीटर दूध और पानी के मिश्रण के साथ मिलाया जाता है तथा अंमति मिश्रण को पुनः 1:
1 के अनुपात वाले 12 लीटर दूध और पानी के मिश्रण के साथ मिलाया जाता है । अंमति
मिश्रण िें दूध और पानी का अनुपात मकतना है ?
(a) 2:3
(b) 1:2
(c) 1:1
(d) 3:1
(e) 2:5
Amount of milk in the final mixture = 42 * 4/7 + 18 * 1/3 + 12 * 1/2 = 36 liters
And, amount of water in the final mixture = 42 * 3/7 + 18 * 2/3 + 12 * 1/2 = 36 liters
So, required ratio = 36: 36 = 1: 1
FOLLOW ON UNACADEMY “ARUN SINGH RAWAT” FOR FREE CLASSES
Top
बिलकुल ऐसा ही आएगा | 5050 Arithmetic
Number SeriesQuestions
| 5 Marks पक्के करो
A book is marked 80% above cost price and sold at 40% discount, if
difference between discount given and profit earned is Rs.460.8, then
find M.R.P. of book?
एक पुस्तक को इसके क्रय िूल्य से 80% अमधक पर अंमकत मकया जाता है और 40%
की छूट पर बेचा जाता है । यमद मदए गए छूट और अमजषत लाभ के बीच का अंतर
460.8 रु. है , तो पुस्तक का अमधकति खुदरा िूल्य(M.R.P.) ज्ञात कीमजए।
Rs.720
Rs.1200
Rs.1296
Rs. 777.6
Rs.1660
Solution:
Let cost price of book = Rs. 100a
180
MRP of book = 100𝑎 × 100 = Rs. 180a
60
Selling price of book = 180𝑎 × 100 = Rs. 108a
ATQ
180𝑎 − 108𝑎 − 108𝑎 − 100𝑎 = 460.8
72𝑎 − 8𝑎 = 460.8
64𝑎 = 460.8
460.8
𝑎= 64
𝑎 = 7.2
So, 180𝑎 = Rs. 1296
FOLLOW ON UNACADEMY “ARUN SINGH RAWAT” FOR FREE CLASSES
Top
बिलकुल ऐसा ही आएगा | 5050 Arithmetic
Number SeriesQuestions
| 5 Marks पक्के करो
A and B together can complete 4/7 th of a work in 16 days. If efficiency
of B is 75% of that of A, then in how many days A alone will complete
the whole work.
A और B मिलकर एक कायष के 4/7 वें भाग को 16 मदनों िें पूरा कर सकते हैं । यमद B
की कायषक्षिता, A का 75% है , तो A अकेले मकतने मदनों िें पूणष कायष को पूरा करे गा।
42 days
36 days
52 days
49 days
55 days
Solution:
Time taken by A and B together to complete the whole work
7
= 16 × 4 = 28 days
Let us assume the efficiency of A = 4x units/day
So, efficiency of B = 3x units/day
So, efficiency of A and B together = 7𝑥 units/day
Let us assume that a alone will complete the whole work in
T days
ATQ
28 × 7𝑥 = 𝑇 × 4𝑥
𝑇 = 49 days
FOLLOW ON UNACADEMY “ARUN SINGH RAWAT” FOR FREE CLASSES
Top
बिलकुल ऐसा ही आएगा | 5050 Arithmetic
Number SeriesQuestions
| 5 Marks पक्के करो
CP of article A is 80% of that of article B. Article A is marked 60%
above its CP and 20% discount is allowed on it. If discount allowed on
article A is Rs.20 more than profit earned on article A, then find CP of
article B.
वस्तु A का सीपी, वस्तु B के सीपी का 80% है । वस्तु A के िूल्य को इसके सीपी से
60% अमधक बढ़ाया जाता है और इस पर 20% की छूट दी जाती है । यमद वस्तु A पर
दी गई छूट, वस्तु A पर अमजषत लाभ से 20 रुपये अमधक है , तो वस्तु B का सीपी ज्ञात
कीमजये।
Rs.600
Rs.650
Rs.675
Rs.575
Rs.625
Solution:
Let cost price of Article – A be Rs. 100x
160
So, marked price of article – A = 100𝑥 × 100 = 𝑅𝑠. 160𝑥
80
And, selling price of article – A = 160𝑥 ×
100
= Rs. 128x
ATQ
160𝑥 − 128𝑥 − 128𝑥 − 100𝑥 = 20
100
𝑥 = 5 Now, CP of article – B = 100 × 5 × = Rs. 625
80
FOLLOW ON UNACADEMY “ARUN SINGH RAWAT” FOR FREE CLASSES
Top
बिलकुल ऐसा ही आएगा | 5050 Arithmetic
Number SeriesQuestions
| 5 Marks पक्के करो
The ratio of efficiency of P & Q is 2:3 respectively and the efficiency of R
is 50% more than that of Q. If P & Q together can complete a work in 13
days, then find in how many days P & R together can complete the
same work?
P और Q की कायषक्षिता का अनुपात क्रिशः 2: 3 है और R की कायषक्षिता Q की
तुलना िें 50% अमधक है । यमद P और Q मिलकर एक कायष को 13 मदनों िें पूरा कर
सकते हैं , तो P और R मिलकर सिान कायष को मकतने मदनों िें पूरा कर सकते हैं ?
12 days
14 days
18 days
10 days
8 days
Solution:
Let efficiency of P & Q are 2x unit/day & 3x unit/day
respectively.
150
So, the efficiency of R = 100 × 3𝑥 = 4.5𝑥
ATQ
Total work = 2𝑥 × 3𝑥 × 13 = 64𝑥 unit
65𝑥
Required time = 2𝑥+4.5𝑥 = 10 days
FOLLOW ON UNACADEMY “ARUN SINGH RAWAT” FOR FREE CLASSES
Top
बिलकुल ऐसा ही आएगा | 5050 Arithmetic
Number SeriesQuestions
| 5 Marks पक्के करो
The ratio of the monthly income of P and Q is 6:5 respectively and the
monthly expenditure of P is 40% more than that of Q . If the saving of P
is 10% less than that of Q , then find what portion of monthly salary is
spent by Q ?
P और Q की िामसक आय का अनुपात क्रिशः 6: 5 है और P का िामसक व्यय, Q
की तुलना िें 40% अमधक है । यमद P की बचत, Q की तुलना िें 10% कि है , तो ज्ञात
कीमजए मक िामसक वेतन का मकतना भाग Q द्वारा खचष मकया जाता है ?
5/8
5/6
3/4
3/5
None of the above.
Solution:
Let monthly income of P and Q be Rs. 6x & Rs. 5x respectively.
And, let monthly expenditure of Q be Rs. 5y
140
So, monthly expenditure of P = 5𝑦 × 100
= Rs. 7y
Hence saving of Q = Rs.(5𝑥 − 5𝑦)
And savings of P = Rs. (6x – 7y)
ATQ
6𝑥−7𝑦 90
= 100
5𝑥−5𝑦

60𝑥 − 70𝑦 = 45𝑥 − 45𝑦


15𝑥 = 25𝑦
𝑥 5
𝑦
=3

Now, let x & y be 5a & 3a respectively .


5×3𝑎 3
Required portion = 5×5𝑎 = 5
FOLLOW ON UNACADEMY “ARUN SINGH RAWAT” FOR FREE CLASSES
Top
बिलकुल ऐसा ही आएगा | 5050 Arithmetic
Number SeriesQuestions
| 5 Marks पक्के करो
A boat covers 300 km downstream in 12 hours and 135 km upstream in
15 hours. If speed of boat in still water is increased by 20%, then find
the time taken by boat to travel 142 km downstream.
एक नाव 12 घण्टे िें 300 मकिी धारा के अनुकूल और 15 घण्टे िें 135 मकिी धारा के
प्रमतकूल दूरी तय करती है । यमद द्धस्थर जल िें नाव की गमत िें 20% की वृद्धद्ध हुई है ,
तब 142 मकिी धारा के अनुकूल यात्रा करने के मलए नाव द्वारा मलया गया सिय ज्ञात
कीमजए।
(a) 4 hours
(b) 10 hours
(c) 7 hours
(d) 5 hours
(e) 9 hours
Downstream speed = 300/12 = 25 km/hr
Upstream speed = 135/15 = 9 km/hr
Speed of boat in still water = (25 + 9)/2 = 17 km/hr
Stream speed = (25 - 9)/2 = 8 km/hr
New speed of boat in still water = 120% of 17 = 20.4 km/hr
New downstream speed = 20.4 + 8 = 28.4 km/hr
Time taken to travel 142 km downstream = 142/28.4 = 5 hours
FOLLOW ON UNACADEMY “ARUN SINGH RAWAT” FOR FREE CLASSES
Top
बिलकुल ऐसा ही आएगा | 5050 Arithmetic
Number SeriesQuestions
| 5 Marks पक्के करो
Sam invested Rs.6,000 & Rs.4,000 in schemes A & B respectively for 5
years. Scheme A offers (R + 4) % p.a. at SI and scheme B offers R% p.a.
at SI. If the total interest received by Sam from both schemes is
Rs.11,200, then find R.
सैि ने 5 वषों के मलए स्कीि A और B िें क्रिशः 6,000 रु. और 4,000 रु. का मनवेश
मकया। स्कीि A साधारण ब्याज पर (R + 4) % वामषषक दर की पेशकश करती है और
स्कीि B साधारण ब्याज पर R% वामषषक की पेशकश करती है । यमद दोनों स्कीि से
सैि द्वारा प्राप्त कुल ब्याज 11,200 रु. है , तो R ज्ञात कीमजए।
12
18
16
24
20
Solution:
ATQ
6000× 𝑅+4 ×5 4000×𝑅×5
+ = 11200
100 100
300𝑅 + 1200 + 200𝑅 = 11200
𝑅 = 20
FOLLOW ON UNACADEMY “ARUN SINGH RAWAT” FOR FREE CLASSES
Top
बिलकुल ऐसा ही आएगा | 5050 Arithmetic
Number SeriesQuestions
| 5 Marks पक्के करो
‘A’ starts a business by investing Rs.6,000. After 6 months, B & C joined
the business by investing Rs.8,000 & Rs.15,000 respectively. After
another 3 months A withdraws Rs.2,000 and at the end of the year, if
total profit is Rs.85,000, then find difference between profit share of A &
B.
‘A’, 6,000 रुपये का मनवेश करके एक व्यवसाय शुरू करता है । 6 िहीने के बाद, B
और C क्रिशः 8,000 रु और 15,000 रु का मनवेश करके व्यवसाय िें शामिल हो जाते
हैं । अन्य 3 िहीने के बाद A, 2,000 रुपये मनकालता है और वषष के अंत िें , यमद कुल
लाभ 85,000 रुपये है , तो A और B के लाभ के महस्े के िध्य अंतर ज्ञात कीमजए।
Rs.4,500
Rs.6,000
Rs.7,500
Rs.9,000
None of the above.
Solution:
Profit sharing ratio of A, B & C
= 6000 × 9 + 6000 − 2000 × 3 ∶ 8000 × 6 : 15000 × 6
= 66000 ∶ 48000 ∶ 90000
= 11 ∶ 8 ∶ 15
ATQ
11−8
Required amount = 85000 × 11+8+15
= 7500 Rs.
FOLLOW ON UNACADEMY “ARUN SINGH RAWAT” FOR FREE CLASSES
Top
बिलकुल ऐसा ही आएगा | 5050 Arithmetic
Number SeriesQuestions
| 5 Marks पक्के करो
A certain amount is invested at 15% SI for 4 years in a scheme and after
4 years interest received from that scheme is invested in another
scheme for 3 years at 20% annual CI. If CI received after 2 years is
Rs.4620, then how much amount is invested in first scheme?
एक योजना िें 4 वषों के मलए 15% साधारण ब्याज पर एक मनमित धनरामश का मनवेश
मकया जाता है और 4 साल के बाद उस योजना से प्राप्त ब्याज को दूसरी योजना िें
20% वामषषक चक्रवृद्धद्ध ब्याज पर 3 साल के मलए मनवेश मकया जाता है । यमद 2 साल के
बाद प्राप्त चक्रवृद्धद्ध ब्याज 4620 रुपये है , तो पहली योजना िें मकतनी धनरामश का
मनवेश मकया जाता है ?
(a) Rs. 15000

(b) Rs. 22500

(c) Rs. 17500

(d) Rs. 20000

(e) Rs. 27500


Let amount invested in first scheme = 100x
SI after 4 years at 15% rate = (100x * 15 * 4)/100 = 60x
CI after 2 years at 20% annual rate = 60x * [(1.2)2 - 1] = 4620
26.4x = 4620
x = 175
Amount invested in first scheme = 100x = Rs.17500
FOLLOW ON UNACADEMY “ARUN SINGH RAWAT” FOR FREE CLASSES
Top
बिलकुल ऐसा ही आएगा | 5050 Arithmetic
Number SeriesQuestions
| 5 Marks पक्के करो
Speed of boat A and B in still water are in the ratio 7: 11 respectively
and their difference is 12 km/hr. If boat B covers 258 km upstream in
12.9 hours, then find the respective ratio of upstream speed of boat A
and B.
द्धस्थर पानी िें नाव A और B की गमत क्रिशः 7 : 11 के अनुपात िें है तथा उनके िध्य
का अंतर 12 मकिी/घण्टा है । यमद नाव B, धारा के प्रमतकूल 258 मकिी की दूरी 12.9
घण्टों िें तय करती है , तब धारा के प्रमतकूल नाव A और B की गमत का सम्बद्धित
अनुपात ज्ञात कीमजए।
(a) 1 : 3

(b) 2 : 5

(c) 3 : 5

(d) 1 : 2

(e) 3 : 4
Here, speed of boat A in still water = 12 * 7/(11 - 7) = 21 km/hr
And, speed of boat B in still water = 21 + 12 = 33 km/hr
Now, upstream speed of boat B = 258/12.9 = 20 km/hr
Then, stream speed = 33 - 20 = 13 km/hr
And, upstream speed of boat A = 21 - 13 = 8 km/hr
Therefore, respective ratio of upstream speed of boat A and B = 8: 20 = 2: 5
FOLLOW ON UNACADEMY “ARUN SINGH RAWAT” FOR FREE CLASSES
Top
बिलकुल ऐसा ही आएगा | 5050 Arithmetic
Number SeriesQuestions
| 5 Marks पक्के करो
Six years ago, age of P was 75% of age of Q at that time and sum of
present age of Q and present age of R is 56 years. If R is two years
older than P, then find sum of present age of P and present age of R?
छह वषष पहले, P की आयु, उस सिय Q की आयु का 75% थी और Q की वतषिान
आयु और R की वतषिान आयु का योग 56 वषष है । यमद R, P से दो वषष बड़ा है , तो P
की वतषिान आयु और R की वतषिान आयु का योग ज्ञात कीमजए।
48 years
50 years
42 years
56 years
54 years
Solution:
Let age of Q six years ago was =4x years
75
so, age of P six years ago was = 4𝑥 × 100 = 3𝑥 years
Present age of R = 56 − 4𝑥 + 6 = (50 − 4𝑥) years
ATQ
50 − 4𝑥 − 3𝑥 + 6 = 2
7𝑥 = 42
𝑥=6
Required sum = 3 × 6 + 6 + 50 − 4 × 6
= 24 + 26 = 50 years
FOLLOW ON UNACADEMY “ARUN SINGH RAWAT” FOR FREE CLASSES
Top
बिलकुल ऐसा ही आएगा | 5050 Arithmetic
Number SeriesQuestions
| 5 Marks पक्के करो
Time taken by P alone to complete a work is 25% more than the time taken by
Q alone to complete the same work. Both starts working together and after 16
days, 10% of the work is remaining. Find the difference between time taken by
P alone and Q alone to complete the same work.
मकसी कायष को पूरा करने के मलए P द्वारा अकेले मलया गया सिय, उसी कायष को अकेले पूरा
करने के मलए Q द्वारा मलए गए सिय से 25% अमधक है । दोनों एक साथ कायष करना शुरू
करते हैं और 16 मदनों के बाद, 10% कायष शेष है । उसी कायष को पूरा करने के मलए अकेले P
और Q द्वारा मलए गए सिय के बीच अंतर ज्ञात कीमजए।
18 days
8 days
10 days
12 days
16 days
Solution:
Let time taken by Q alone to complete the whole wok be 4x days
125
So, time taken by P alone completer the whole wok = 4𝑥 × 100 = 5𝑥 days
ATQ
16 16 100−10
+ 5𝑥 =
4𝑥 100
80+64 9
= 10
20𝑥
20𝑥 = 160
𝑥 = 8 days
Required difference = 5𝑥 − 4𝑥 = 𝑥 = 8 𝑑𝑎𝑦𝑠
FOLLOW ON UNACADEMY “ARUN SINGH RAWAT” FOR FREE CLASSES
Top
बिलकुल ऐसा ही आएगा | 5050 Arithmetic
Number SeriesQuestions
| 5 Marks पक्के करो
A shopkeeper sold a table at 40% discount and thus making a loss of
10%. If he gives a discount equals to half of the previous discount then
he makes a profit of Rs. 60. Find at what price shopkeeper should sell
the table to make a profit of 25%?
एक दुकानदार ने 40% छूट पर एक टे बल बेची और इस प्रकार उसे 10% की हामन
हुई। यमद वह मपछली छूट के आधे के बराबर छूट दे ता है तो उसे 60 रु का लाभ
अमजषत होता है । 25% का लाभ किाने के मलए दुकानदार को टे बल मकस कीित पर
बेचनी चामहए?
Rs. 600
Rs. 540
Rs. 375
Rs. 360
Rs. 480
Solution:
Let mark price of table = Rs. 100x
100𝑥×60
After discount of 40% of selling price of table = = Rs. 60x
100
Given loss on selling table at 405 discount = 10%
60𝑥 200
Cost price of table = × 100 = Rs. 𝑥 ..(i)
90 3
When table is sold at half discount i.e. 20% discount then:
100𝑥×80
Selling price of table = = Rs.80x
100
Cost price of table = 80𝑥 − 60 ..(ii)
Comparing (i) & (ii)
200𝑥
= 80𝑥 − 60
3
𝑥 = 4.5 Rs.
200
Cost price of table = × 4.5 = Rs.300
3
125
So, required selling price = 300 × 100 = 375 Rs
FOLLOW ON UNACADEMY “ARUN SINGH RAWAT” FOR FREE CLASSES
Top
बिलकुल ऐसा ही आएगा | 5050 Arithmetic
Number SeriesQuestions
| 5 Marks पक्के करो
A 360 meters long passenger train crosses a goods train of length 540 m in
21.6 seconds when running towards each other. If the speed of goods train
is 50% more than that of passenger train, then find time taken by the
goods train to cross passenger train when running in same direction.
एक 360 िीटर लंबी यात्री टर े न एक दूसरे की ओर दौड़ने पर 21.6 सेकंर् िें 540 िीटर
लंबी िालगाड़ी को पार करती है । यमद िालगाड़ी की गमत, यात्री टर े न की तुलना िें 50%
अमधक लंबी है , तो उसी मदशा िें चलने पर यात्री टर े न को पार करने के मलए िालगाड़ी द्वारा
मलया गया सिय ज्ञात कीमजए।
144 sec
72 sec
54 sec
102 sec
108 sec
Solution:
Let speed of passenger train = 2𝑥 𝑚/𝑠𝑒𝑐
So, speed of goods train = 3𝑥 𝑚/𝑠𝑒𝑐
ATQ
360+540
2𝑥 + 3𝑥 = 21.6
125
5𝑥 = m/s
3
25
𝑥= m/s
3
360+540
Required time = 3𝑥−2𝑥
900
= × 3 = 108 seconds
25
FOLLOW ON UNACADEMY “ARUN SINGH RAWAT” FOR FREE CLASSES
Top
बिलकुल ऐसा ही आएगा | 5050 Arithmetic
Number SeriesQuestions
| 5 Marks पक्के करो
A man invested Rs. X at the rate of R% p.a. on compound interest
compounding annually. If compound interest received in second year
and third year is Rs.2400 and Rs.2880 respectively, then find the value
of X.
एक व्यद्धि ने वामषषक रूप से संयोमजत चक्रवृद्धद्ध ब्याज पर R% प्रमत वषष की दर से X
रु का मनवेश मकया। यमद दूसरे वषष और तीसरे वषष िें प्राप्त चक्रवृद्धद्ध ब्याज क्रिशः
2400 रुपये और 2880 रुपये है , तो X का िान ज्ञात कीमजए।
Rs.20000
Rs.10000
Rs.12000
Rs.15000
None of the above
Solution:
ATQ
3𝑟𝑑 𝑦𝑒𝑎𝑟 𝐶𝐼−2𝑛𝑑 𝑦𝑒𝑎𝑟 𝐶𝐼
𝑅% = × 100
2𝑛𝑑 𝑦𝑒𝑎𝑟𝑠 𝐶𝐼
2800−2400
= × 100
2400
480
= 2400 × 100 = 20%
Now,
𝑅 2 𝑋×𝑅×1
𝑋 1+ −𝑋 − = 2400
100 100

20 2 𝑋×20
𝑋 1+ −𝑋− = 2400
100 100

𝑋 = 10000 Rs.
FOLLOW ON UNACADEMY “ARUN SINGH RAWAT” FOR FREE CLASSES
Top
बिलकुल ऐसा ही आएगा | 5050 Arithmetic
Number SeriesQuestions
| 5 Marks पक्के करो
A bag contains 3 blue balls, 5 white balls and 4 red balls. If 2 balls are
picked randomly from the bag, then find the probability of getting both
balls of the same color.
एक थैले िें 3 नीली गेंदे, 5 सफ़ेद गेंदे और 4 लाल गेंदे हैं । यमद 2 गेंदों को यादृद्धिक
रूप से मनकाला जाता है , तो दोनों गेंदे सिान रं ग की मनकालने की प्रामयकता ज्ञात
कीमजये।
8/33
5/22
19/66
17/66
3/11
Solution:
Possible cases = (2 blue balls) or (2 red balls) or (2 white
balls)
3𝐶 5𝐶 4𝐶
Required probability = 12𝐶
2
+ 12𝐶
2
+ 12𝐶
2
2 2 2
19
= 66
FOLLOW ON UNACADEMY “ARUN SINGH RAWAT” FOR FREE CLASSES
Top
बिलकुल ऐसा ही आएगा | 5050 Arithmetic
Number SeriesQuestions
| 5 Marks पक्के करो
In an election, there were two candidates P & Q from the same
constituency. The total no. of votes in their constituency was 30,000 and
30% of the total votes were not polled. If 60% of the total number of
votes polled went in the favor of P, then find difference between total
votes received by P & Q .
एक चुनाव िें, एक ही मनवाषचन क्षेत्र से दो उम्मीदवार P और Q थे। उनके मनवाषचन
क्षेत्र िें ितों की कुल संख्या 30,000 थी और कुल ितों के 30% ने ितदान नही ं मकया
गया था। यमद ितदान के कुल ितों का 60% P के पक्ष िें गया, तो P और Q को मिले
कुल ितों के बीच अंतर ज्ञात कीमजए।
6000
5400
4000
4200
4800
Solution:
Given total votes = 30000
100−30
Total polled votes = 30000 × = 21000
100
60
Votes cast in favor of P = 21000 × 100 = 12600
Votes cast in favor of Q = 21000 − 12600 = 8400
Required difference = 12600 − 8400 = 4200
FOLLOW ON UNACADEMY “ARUN SINGH RAWAT” FOR FREE CLASSES
Top
बिलकुल ऐसा ही आएगा | 5050 Arithmetic
Number SeriesQuestions
| 5 Marks पक्के करो
In a class there are 60 girls and 30 boys, and the total average weight of
𝟐
class is 48 kg If the average weight of boys is 56 kg, then find the
𝟑
difference between the average weight of one boy and one girl?
𝟐
एक कक्षा िें 60 लड़मकयां और 30 लड़के हैं और कक्षा का कुल औसत भार 48
𝟑
मक.ग्रा. है । यमद लड़कों का औसत भार 56 मक.ग्रा. है , तो एक लड़के और एक लड़की
के औसत भार के बीच का अंतर ज्ञात कीमजए।
11 kg
9 kg
15 kg
10 kg
None of these
Solution:
146
Total weight of class = 60 + 30 × = 4380 kg
3
Total weight of boys = 30 × 56 = 1680 kg
4380−1680
So, weight of one girl = = 45 kg
60
Required difference = 56 − 45 = 11 kg
FOLLOW ON UNACADEMY “ARUN SINGH RAWAT” FOR FREE CLASSES
Top
बिलकुल ऐसा ही आएगा | 5050 Arithmetic
Number SeriesQuestions
| 5 Marks पक्के करो
A can complete a work in 40 days and B is 25% more efficient than A. B
worked for 12 days on same work and he left, remaining work
completed by Sam in 30 days. Find in how many days Sam can
complete the same work alone?
A 40 मदनों िें एक कायष पूरा कर सकता है और A की तुलना िें B 25% अमधक
कायषकुशल है । B ने सिान कायष पर 12 मदन कायष मकया और वह छोड़ दे ता है , शेष
काि 30 मदनों िें सैि द्वारा पूरा मकया गया। सिान कायष अकेले सै ि मकतने मदनों िें
पूरा कर सकता है ?
40 days
42 days
36 days
24 days
48 days
Solution:
4
B can complete the work alone = 40 × 5 = 32 days
Let us assume Sam can complete the work in ‘d’ days
ATQ-
12 30
+ =1
32 𝑑
30 5
=8
𝑑
30×8
𝑑= = 48 days
5
FOLLOW ON UNACADEMY “ARUN SINGH RAWAT” FOR FREE CLASSES
Top
बिलकुल ऐसा ही आएगा | 5050 Arithmetic
Number SeriesQuestions
| 5 Marks पक्के करो
B invested 37.5% less than the investment of A and C invested 20%
more than the investment of B in a business. If A, B and C invested for 4
months, 8 months and 6 months respectively and the sum of profit
share of B & C is Rs. 17100, then find the profit share of A?
एक व्यवसाय िें B ने A के मनवेश से 37.5% कि मनवेश मकया और C ने B के मनवेश
से 20% अमधक मनवेश मकया। यमद A, B और C क्रिशः 4 िहीने, 8 िहीने और 6
िहीने के मलए मनवेश करता है और B और C के लाभ शेयर का योग 17100 रु. है , तो
A का लाभ महस्ा ज्ञात कीमजए।
Rs. 3600
Rs. 7200
Rs. 6400
Rs. 9600
Rs. 4800
Solution:
Let investment of A = 8x
37.5
So, investment of B = 8𝑥 × 1 − 100 = 5𝑥
6
Investment of C = 5𝑥 × 5 = 6𝑥
Profit ratio of A, B & C = 8𝑥 × 4 : 5𝑥 × 8 : 6𝑥 × 6 = 8 ∶
10 ∶ 9
ATQ
10 + 9 units = 17100
1 unit = 900 Rs.
Profit share of A = 8 × 900 = 7200 Rs.
FOLLOW ON UNACADEMY “ARUN SINGH RAWAT” FOR FREE CLASSES
Top
बिलकुल ऐसा ही आएगा | 5050 Arithmetic
Number SeriesQuestions
| 5 Marks पक्के करो
If the volume of a cone & a sphere is equal and radius of the sphere is
50% of the radius of the cone, then find the ratio of the radius of the
sphere to height of the cone?
यमद एक शंकु और एक गोले का आयतन बराबर है और गोले की मत्रज्या, शंकु की
मत्रज्या का 50% है , तो गोले की मत्रज्या का शंकु की ऊंचाई से अनुपात ज्ञात कीमजए।
1:2
3:2
3:1
1 :1
2 :1
Solution:
Let radius & height of the cone be r cm & h cm respectively.
And let radius of sphere = R cm
Volume of cone = Volume of sphere
1 4
× 𝜋𝑟 2 ℎ = 3 𝜋𝑅3
3
𝑟 2 ℎ = 4𝑅3 …(i)
Given r = 2R
Putting value of r in (i)
2𝑅 2 × ℎ = 4𝑅3
4𝑅2 × ℎ = 4𝑅3
ℎ=𝑅
𝑅 𝑅
So, required ratio = ℎ = 𝑅
=1∶1
FOLLOW ON UNACADEMY “ARUN SINGH RAWAT” FOR FREE CLASSES
Top
बिलकुल ऐसा ही आएगा | 5050 Arithmetic
Number SeriesQuestions
| 5 Marks पक्के करो
𝟏
A shopkeeper marks up the price of an article 33 % above its cost
𝟑
price, and he offers a discount of 20% on it and thus he earn a profit of
Rs. 70. If he had sold the article at 30% profit, then what will be the new
selling price of the article?
𝟏
एक दुकानदार एक वस्तु के िूल्य को उसके क्रय िूल्य से 33 %अमधक बढ़ाता है ,
𝟑
और इस पर 20% की छूट प्रदान करता है और इस प्रकार वह 70 रुपये का लाभ प्राप्त
करता है । यमद वह वस्तु को 30% लाभ पर बेचता है , तो वस्तु का नया मवक्रय िूल्य क्या
होगा?
Rs. 1050
Rs. 1350
Rs. 1400
Rs. 1280
Rs. 1365
Solution:
Let cost price of the article = Rs 300x
300+100
So, marked price of the article = × 300𝑥 = Rs. 400x
300
80
Now, selling price of the article = 100 × 400𝑥
= Rs. 320x
ATQ
320𝑥 − 300𝑥 = 70
𝑥 = 3.5 Rs.
So, cost price of the article = 300 × 3.5 = Rs.1050
130
Required selling price = 1050 × 100 = Rs. 1365
FOLLOW ON UNACADEMY “ARUN SINGH RAWAT” FOR FREE CLASSES
Top
बिलकुल ऐसा ही आएगा | 5050 Arithmetic
Number SeriesQuestions
| 5 Marks पक्के करो
A bag contains (x+1) green balls and (x+4) yellow balls. If one ball is
drawn at random from the bag, then probability of getting a green ball
is 3/7 . find the total number of balls in the bag?
एक बैग िें (x + 1) हरी गेंदें और (x + 4) पीली गेंदें हैं । यमद बैग से एक गेंद को
यादृद्धिक रूप से मनकाला जाता है , तो हरी गेंद प्राप्त करने की सं भावना 3/7 है । बैग
िें गेंदों की कुल संख्या ज्ञात कीमजये।
23
19
21
15
13
Solution:
ATQ
3
Probability of choosing a green ball = 7
𝑥+1 3
=7
𝑥+1+𝑥+4
𝑥+1 3
=7
2𝑥+5
𝑥=8
Now, total number of balls in the bag = 9 + 12 = 21 balls

You might also like